HCAL III Exam 03 Practice Questions

¡Supera tus tareas y exámenes ahora con Quizwiz!

Which preventive actions by the nurse will help limit the development of systemic inflammatory response syndrome (SIRS) in patients admitted to the hospital (select all that apply)? Use aseptic technique when caring for invasive lines or devices. Ambulate postoperative patients as soon as possible after surgery. Remove indwelling urinary catheters as soon as possible after surgery. Advocate for parenteral nutrition for patients who cannot take oral feedings. Administer prescribed antibiotics within 1 hour for patients with possible sepsis.

ANS: A, B, C, E Because sepsis is the most frequent etiology for SIRS, measures to avoid infection such as removing indwelling urinary catheters as soon as possible, use of aseptic technique, and early ambulation should be included in the plan of care. Adequate nutrition is important in preventing SIRS. Enteral, rather than parenteral nutrition is preferred when patients are unable to take oral feedings because enteral nutrition helps maintain the integrity of the intestine, thus decreasing infection risk. Antibiotics should be administered within 1 hour after being prescribed to decrease the risk of sepsis progressing to SIRS. DIF: Cognitive Level: Analyze (analysis) REF: 1605 TOP: Nursing Process: Planning

A patient with suspected neurogenic shock after a diving accident has arrived in the emergency department. A cervical collar is in place. Which actions should the nurse take (select all that apply)? Prepare to administer atropine IV. Obtain baseline body temperature. Infuse large volumes of lactated Ringers solution. Provide high-flow oxygen (100%) by non-rebreather mask. Prepare for emergent intubation and mechanical ventilation.

ANS: A, B, D, E All of the actions are appropriate except to give large volumes of lactated Ringers solution. The patient with neurogenic shock usually has a normal blood volume, and it is important not to volume overload the patient. In addition, lactated Ringers solution is used cautiously in all shock situations because the failing liver cannot convert lactate to bicarbonate. DIF: Cognitive Level: Apply (application) REF: 1602 TOP: Nursing Process: Implementation

An example of a sign of renal failure is a urine output, despite volume resuscitation, of: a. 0.3mL/kg/hr b. 0.6mL/kg/hr c. 0.9 mL/kg/hr d. 1.2mL/kg/hr

ANS: B

Two of the SIRS criteria plus a known or suspected infection is considered which stage is the progression of sepsis? a. Severe sepsis b. Sepsis c. Septic shock d. Presepsis

ANS: B

24. After change-of-shift report in the progressive care unit, who should the nurse care for first? Patient who had an inferior myocardial infarction 2 days ago and has crackles in the lung bases Patient with suspected urosepsis who has new orders for urine and blood cultures and antibiotics Patient who had a T5 spinal cord injury 1 week ago and currently has a heart rate of 54 beats/minute Patient admitted with anaphylaxis 3 hours ago who now has clear lung sounds and a blood pressure of 108/58 mm Hg

ANS: B Antibiotics should be administered within the first hour for patients who have sepsis or suspected sepsis in order to prevent progression to systemic inflammatory response syndrome (SIRS) and septic shock. The data on the other patients indicate that they are more stable. Crackles heard only at the lung bases do not require immediate intervention in a patient who has had a myocardial infarction. Mild bradycardia does not usually require atropine in patients who have a spinal cord injury. The findings for the patient admitted with anaphylaxis indicate resolution of bronchospasm and hypotension. DIF: Cognitive Level: Analyze (analysis) REF: 1600 OBJ: Special Questions: Prioritization; Multiple Patients TOP: Nursing Process: Assessment

9. Which finding is the best indicator that the fluid resuscitation for a patient with hypovolemic shock has been effective? a. Hemoglobin is within normal limits. b. Urine output is 60 mL over the last hour. c. Central venous pressure (CVP) is normal. d. Mean arterial pressure (MAP) is 72 mm Hg.

ANS: B Assessment of end organ perfusion, such as an adequate urine output, is the best indicator that fluid resuscitation has been successful. The hemoglobin level, CVP, and MAP are useful in determining the effects of fluid administration, but they are not as useful as data indicating good organ perfusion. DIF: Cognitive Level: Apply (application) REF: 1598 TOP: Nursing Process: Evaluation

27. Which information obtained by the nurse caring for a patient with thrombocytopenia should be immediately communicated to the health care provider? a. The platelet count is 52,000/L. b. The patient is difficult to arouse. c. There are purpura on the oral mucosa. d. There are large bruises on the patients back.

ANS: B Difficulty in arousing the patient may indicate a cerebral hemorrhage, which is life threatening and requires immediate action. The other information should be documented and reported but would not be unusual in a patient with thrombocytopenia. DIF: Cognitive Level: Apply (application) REF: 625 OBJ: Special Questions: Prioritization TOP: Nursing Process: Assessment

22. The following interventions are ordered by the health care provider for a patient who has respiratory distress and syncope after eating strawberries. Which will the nurse complete first? Start a normal saline infusion. Give epinephrine (Adrenalin). Start continuous ECG monitoring. Give diphenhydramine (Benadryl).

ANS: B Epinephrine rapidly causes peripheral vasoconstriction, dilates the bronchi, and blocks the effects of histamine and reverses the vasodilation, bronchoconstriction, and histamine release that cause the symptoms of anaphylaxis. The other interventions are also appropriate but would not be the first ones completed. DIF: Cognitive Level: Apply (application) REF: 1601 OBJ: Special Questions: Prioritization TOP: Nursing Process: Implementation

A patient is experiencing pulmonary edema as an exacerbation of chronic left-sided heart failure. The nurses assesses the patient for which of the following manifestations? a. Weight loss b. Bilateral crackles c. Distended neck veins d. Peripheral pitting edema

ANS: B Pulmonary edema is the basis of ARDS - we see a component of pulmonary edema when pt goes into ARDS.

10. Which intervention will the nurse include in the plan of care for a patient who has cardiogenic shock? a. Check temperature every 2 hours. b. Monitor breath sounds frequently. c. Maintain patient in supine position. d. Assess skin for flushing and itching.

ANS: B Since pulmonary congestion and dyspnea are characteristics of cardiogenic shock, the nurse should assess the breath sounds frequently. The head of the bed is usually elevated to decrease dyspnea in patients with cardiogenic shock. Elevated temperature and flushing or itching of the skin are not typical of cardiogenic shock. DIF: Cognitive Level: Apply (application) REF: 1589 TOP: Nursing Process: Implementation

5. After receiving 2 L of normal saline, the central venous pressure for a patient who has septic shock is 10 mm Hg, but the blood pressure is still 82/40 mm Hg. The nurse will anticipate an order for a. nitroglycerine (Tridil). b. norepinephrine (Levophed). c. sodium nitroprusside (Nipride). d. methylprednisolone (Solu-Medrol).

ANS: B When fluid resuscitation is unsuccessful, vasopressor drugs are administered to increase the systemic vascular resistance (SVR) and blood pressure, and improve tissue perfusion. Nitroglycerin would decrease the preload and further drop cardiac output and BP. Methylprednisolone (Solu-Medrol) is considered if blood pressure does not respond first to fluids and vasopressors. Nitroprusside is an arterial vasodilator and would further decrease SVR. DIF: Cognitive Level: Apply (application) REF: 1599 TOP: Nursing Process: Planning

17. The nurse is caring for a patient who has septic shock. Which assessment finding is most important for the nurse to report to the health care provider? a. Blood pressure (BP) 92/56 mm Hg b. Skin cool and clammy c. Oxygen saturation 92% d. Heart rate 118 beats/minute

ANS: B Because patients in the early stage of septic shock have warm and dry skin, the patients cool and clammy skin indicates that shock is progressing. The other information will also be reported, but does not indicate deterioration of the patients status. DIF: Cognitive Level: Apply (application) REF: 1594 OBJ: Special Questions: Prioritization TOP: Nursing Process: Assessment

7. A patient with cardiogenic shock has the following vital signs: BP 102/50, pulse 128, respirations 28. The pulmonary artery wedge pressure (PAWP) is increased and cardiac output is low. The nurse will anticipate an order for which medication? a. 5% human albumin b. Furosemide (Lasix) IV c. Epinephrine (Adrenalin) drip d. Hydrocortisone (Solu-Cortef)

ANS: B The PAWP indicates that the patients preload is elevated, and furosemide is indicated to reduce the preload and improve cardiac output. Epinephrine would further increase heart rate and myocardial oxygen demand. 5% human albumin would also increase the PAWP. Hydrocortisone might be considered for septic or anaphylactic shock. DIF: Cognitive Level: Apply (application) REF: 1601 TOP: Nursing Process: Planning

A patient recovering from second-and third degree burns over 30% of his body and is now ready for discharge. The first action the nurse should take when meeting with the patient would be to? a. Arrange a return-to-the clinic appointment and prescription for pain medication. b. Teach the patient and the caregiver proper wound care to be performed at home. c. Review the patient's current health care status and readiness for discharge to home. d. Give the patient written discharge information and websites for additional information for burn survivors.

ANS: C

Septic shock is caused by which source? a. Allergens b. Loss of sympathetic tone c. Microorganisms d. Non-immunologic activation of mast cells and basophils

ANS: C

Which kind of shock occurs from inadequate fluid volume in the intravascular space? A. Anaphylactic B. Cardiogenic C. Hypovolemic D. Neurogenic

ANS: C

8. The emergency department (ED) nurse receives report that a patient involved in a motor vehicle crash is being transported to the facility with an estimated arrival in 1 minute. In preparation for the patients arrival, the nurse will obtain a. hypothermia blanket. b. lactated Ringers solution. c. two 14-gauge IV catheters. d. dopamine (Intropin) infusion.

ANS: C A patient with multiple trauma may require fluid resuscitation to prevent or treat hypovolemic shock, so the nurse will anticipate the need for 2 large bore IV lines to administer normal saline. Lactated Ringers solution should be used cautiously and will not be ordered until the patient has been assessed for possible liver abnormalities. Vasopressor infusion is not used as the initial therapy for hypovolemic shock. Patients in shock need to be kept warm not cool. DIF: Cognitive Level: Apply (application) REF: 1597 TOP: Nursing Process: Planning

23. Which finding about a patient who is receiving vasopressin (Pitressin) to treat septic shock is most important for the nurse to communicate to the health care provider? The patients urine output is 18 mL/hr. The patients heart rate is 110 beats/minute. The patient is complaining of chest pain. The patients peripheral pulses are weak.

ANS: C Because vasopressin is a potent vasoconstrictor, it may decrease coronary artery perfusion. The other information is consistent with the patients diagnosis and should be reported to the health care provider but does not indicate a need for a change in therapy. DIF: Cognitive Level: Apply (application) REF: 1599 OBJ: Special Questions: Prioritization TOP: Nursing Process: Assessment

3. A 19-year-old patient with massive trauma and possible spinal cord injury is admitted to the emergency department (ED). Which assessment finding by the nurse will help confirm a diagnosis of neurogenic shock? a. Inspiratory crackles. b. Cool, clammy extremities. c. Apical heart rate 45 beats/min. d. Temperature 101.2 F (38.4 C).

ANS: C Neurogenic shock is characterized by hypotension and bradycardia. The other findings would be more consistent with other types of shock. DIF: Cognitive Level: Understand (comprehension) REF: 1590 TOP: Nursing Process: Assessment

4. An older patient with cardiogenic shock is cool and clammy and hemodynamic monitoring indicates a high systemic vascular resistance (SVR). Which intervention should the nurse anticipate doing next? a. Increase the rate for the dopamine (Intropin) infusion. b. Decrease the rate for the nitroglycerin (Tridil) infusion. c. Increase the rate for the sodium nitroprusside (Nipride) infusion. d. Decrease the rate for the 5% dextrose in normal saline (D5/.9 NS) infusion.

ANS: C Nitroprusside is an arterial vasodilator and will decrease the SVR and afterload, which will improve cardiac output. Changes in the D5/.9 NS and nitroglycerin infusions will not directly decrease SVR. Increasing the dopamine will tend to increase SVR. DIF: Cognitive Level: Apply (application) REF: 1600 TOP: Nursing Process: Planning

21. The patient with neurogenic shock is receiving a phenylephrine (Neo-Synephrine) infusion through a right forearm IV. Which assessment finding obtained by the nurse indicates a need for immediate action? The patients heart rate is 58 beats/minute. The patients extremities are warm and dry. The patients IV infusion site is cool and pale. The patients urine output is 28 mL over the last hour.

ANS: C The coldness and pallor at the infusion site suggest extravasation of the phenylephrine. The nurse should discontinue the IV and, if possible, infuse the medication into a central line. An apical pulse of 58 is typical for neurogenic shock but does not indicate an immediate need for nursing intervention. A 28-mL urinary output over 1 hour would require the nurse to monitor the output over the next hour, but an immediate change in therapy is not indicated. Warm, dry skin is consistent with early neurogenic shock, but it does not indicate a need for a change in therapy or immediate action. DIF: Cognitive Level: Apply (application) REF: 1599 OBJ: Special Questions: Prioritization TOP: Nursing Process: Assessment

20. A patient who has been involved in a motor vehicle crash arrives in the emergency department (ED) with cool, clammy skin; tachycardia; and hypotension. Which intervention ordered by the health care provider should the nurse implement first? Insert two large-bore IV catheters. Initiate continuous electrocardiogram (ECG) monitoring. Provide oxygen at 100% per non-rebreather mask. Draw blood to type and crossmatch for transfusions.

ANS: C The first priority in the initial management of shock is maintenance of the airway and ventilation. ECG monitoring, insertion of IV catheters, and obtaining blood for transfusions should also be rapidly accomplished but only after actions to maximize oxygen delivery have been implemented. DIF: Cognitive Level: Apply (application) REF: 1597 OBJ: Special Questions: Prioritization TOP: Nursing Process: Implementation

25. After reviewing the information shown in the accompanying figure for a patient with pneumonia and sepsis, which information is most important to report to the health care provider? Temperature and IV site appearance Oxygen saturation and breath sounds Platelet count and presence of petechiae Blood pressure, pulse rate, respiratory rate.

ANS: C The low platelet count and presence of petechiae suggest that the patient may have disseminated intravascular coagulation and that multiple organ dysfunction syndrome (MODS) is developing. The other information will also be discussed with the health care provider but does not indicate that the patients condition is deteriorating or that a change in therapy is needed immediately. DIF: Cognitive Level: Analyze (analysis) REF: 1596 OBJ: Special Questions: Prioritization TOP: Nursing Process: Assessment

36. A patient with possible disseminated intravascular coagulation arrives in the emergency department with a blood pressure of 82/40, temperature 102 F (38.9 C), and severe back pain. Which physician order will the nurse implement first? a. Administer morphine sulfate 4 mg IV. b. Give acetaminophen (Tylenol) 650 mg. c. Infuse normal saline 500 mL over 30 minutes. d. Schedule complete blood count and coagulation studies.

ANS: C The patients blood pressure indicates hypovolemia caused by blood loss and should be addressed immediately to improve perfusion to vital organs. The other actions also are appropriate and should be rapidly implemented, but improving perfusion is the priority for this patient. DIF: Cognitive Level: Apply (application) REF: 633 OBJ: Special Questions: Prioritization TOP: Nursing Process: Implementation

12. A nurse is assessing a patient who is receiving a nitroprusside (Nipride) infusion to treat cardiogenic shock. Which finding indicates that the medication is effective? a. No new heart murmurs b. Decreased troponin level c. Warm, pink, and dry skin d. Blood pressure 92/40 mm Hg

ANS: C Warm, pink, and dry skin indicates that perfusion to tissues is improved. Since nitroprusside is a vasodilator, the blood pressure may be low even if the medication is effective. Absence of a heart murmur and a decrease in troponin level are not indicators of improvement in shock. DIF: Cognitive Level: Apply (application) REF: 1600 TOP: Nursing Process: Evaluation

While performing triage in the emergency department, the nurse determines that which of the following patients should be seen first? a. A patient with a deformed leg indicating a fractured tibia; blood pressure 110/60 mm Hg, pulse 86 beats/min, respirations 18 breaths/min. b. A patient with burns on the face and chest; blood pressure 120/80 mm Hg, pulse 92 beats/min, respirations 24 breaths/min. c. A patient with type 1 diabetes in ketoacidosis; blood pressure 100/60 mm Hg, pulse 100 beats/min, respirations 32 breaths/min. d. A patient with a respiratory infection with a cough productive of greenish sputum; blood pressure 128/86 mm Hg, pulse 88 beats/min, respirations 26 breaths/min.

ANS: C because of low BP and high pulse and kussmaul respirations = shock Disaster Planning PP

A patient admitted to the hospital from a long-term care facility appears to be in the late stage of shock with systemic inflammatory response syndrome (SIRS). Which of the following orders implemented by the nurse has the highest priority? a. Insert an indwelling urinary catheter. b. Insert two large-bore intravenous catheters. c. Administer 0.9% normal saline at 100 mL/hr. d. Administer 100% oxygen by non-rebreather mask.

ANS: D

What occurs during the refractory stage of shock? A. Cardiac output (CO) is decreased, and tissue perfusion is threatened. B. Compensatory mechanisms begin failing. C. Homeostatic mechanisms begin to work. D. Shock becomes unresponsive to therapy.

ANS: D

12. A patient who has been receiving a heparin infusion and warfarin (Coumadin) for a deep vein thrombosis (DVT) is diagnosed with heparin-induced thrombocytopenia (HIT) when her platelet level drops to 110,000/L. Which action will the nurse include in the plan of care? a. Use low-molecular-weight heparin (LMWH) only. b. Administer the warfarin (Coumadin) at the scheduled time. c. Teach the patient about the purpose of platelet transfusions. d. Discontinue heparin and flush intermittent IV lines using normal saline.

ANS: D All heparin is discontinued when the HIT is diagnosed. The patient should be instructed to never receive heparin or LMWH. Warfarin is usually not given until the platelet count has returned to 150,000/L. The platelet count does not drop low enough in HIT for a platelet transfusion, and platelet transfusions increase the risk for thrombosis. DIF: Cognitive Level: Apply (application) REF: 627 TOP: Nursing Process: Planning

When monitoring initial fluid replacement for the patient with 40% TBSA deep partial-thickness and full-thickness burns, which of the following findings is of most concern to the nurse? a. Urine output of 35 ml/hr b. Serum K+ of 4.5 mEq/L c. Decreased bowel sounds d. Blood pressure of 86/72 mm Hg

ANS: D No pulse pressure so no circulation.

6. To evaluate the effectiveness of the pantoprazole (Protonix) ordered for a patient with systemic inflammatory response syndrome (SIRS), which assessment will the nurse perform? a. Auscultate bowel sounds. b. Palpate for abdominal pain. c. Ask the patient about nausea. d. Check stools for occult blood.

ANS: D Proton pump inhibitors are given to decrease the risk for stress ulcers in critically ill patients. The other assessments also will be done, but these will not help in determining the effectiveness of the pantoprazole administration. DIF: Cognitive Level: Apply (application) REF: 1602 TOP: Nursing Process: Evaluation

During the emergent phase of burn injury, the nurse assesses for the presence of hypovolemia. In burns, hypovolemia occurs primarily as a result of a. blood loss from injured tissue. b. third spacing of fluid into fluid-filled vesicles. c. evaporation of fluid from denuded body surfaces. d. capillary permeability with fluid shift to the interstitium.

ANS: D This is why swelling with burns is such a huge complication and needs to be assessed at all times.

Assessment of a male patient during the primary survey indicates delayed capillary refill of the extremities. He cannot explain the events before admission to the ED. Which action should the nurse take immediately? a. Apply leads to the patient's chest to initiate ECG monitoring. b. Insert one or two large-bore IV catheters to start IV fluid resuscitation. c. Continue the primary survey to complete a brief neurologic examination. d. Initiate pulse oximetry by placing a monitoring device on the patient's index finger.

ANS: D because it deals with airway!! B says one OR two; we NEED 2!! Disaster Planning PP

13. Which assessment information is most important for the nurse to obtain to evaluate whether treatment of a patient with anaphylactic shock has been effective? a. Heart rate b. Orientation c. Blood pressure d. Oxygen saturation

ANS: D Because the airway edema that is associated with anaphylaxis can affect airway and breathing, the oxygen saturation is the most critical assessment. Improvements in the other assessments will also be expected with effective treatment of anaphylactic shock. DIF: Cognitive Level: Apply (application) REF: 1602 TOP: Nursing Process: Evaluation

16. When the nurse educator is evaluating the skills of a new registered nurse (RN) caring for patients experiencing shock, which action by the new RN indicates a need for more education? a. Placing the pulse oximeter on the ear for a patient with septic shock b. Keeping the head of the bed flat for a patient with hypovolemic shock c. Increasing the nitroprusside (Nipride) infusion rate for a patient with a high SVR d. Maintaining the room temperature at 66 to 68 F for a patient with neurogenic shock

ANS: D Patients with neurogenic shock may have poikilothermia. The room temperature should be kept warm to avoid hypothermia. The other actions by the new RN are appropriate. DIF: Cognitive Level: Apply (application) REF: 1590 OBJ: Special Questions: Delegation TOP: Nursing Process: Evaluation

15. Which laboratory result will the nurse expect to show a decreased value if a patient develops heparin- induced thrombocytopenia (HIT)? a. Prothrombin time b. Erythrocyte count c. Fibrinogen degradation products d. Activated partial thromboplastin time

ANS: D Platelet aggregation in HIT causes neutralization of heparin, so that the activated partial thromboplastin time will be shorter and more heparin will be needed to maintain therapeutic levels. The other data will not be affected by HIT. DIF: Cognitive Level: Apply (application) REF: 625 TOP: Nursing Process: Assessment

A patient with a gunshot wound to the abdomen is being treated for hypovolemic shock and septic shock. TO monitor the patient for early organ damage associated with MODS, what is most important for the nurse to assess? a. urine output b. breath sounds c. peripheral cirulation d. central venous pressure

B

Which patient manifestations confirm the development of MODS? a. upper GI bleeding, GCS score of 7, and Hct of 25% b. elevated serum bilirubin, serum creatinine of 3.8, and platelet count of 15,000 c. urine output of 30 ml/hr, BUN of 45, and WBC count of 1120 d. respiratory rate of 45, PaCO2 of 60, and chest x-ray with bilateral diffuse patchy infiltrates.

B

The patient has hypovolemic shock. By compensatory mechanisms, the decreased blood pressure (BP) has led to decreased capillary hydrostatic pressure, activation of the sympathetic nervous system and α- and β-adrenergic stimulation, decreased blood flow, and increased cardiac output (CO). In what order should the next pathophysiologic mechanisms of the compensated stage of hypovolemic shock occur? (Answer with a letter followed by a comma and a space (e.g. A, B, C, D).) a. Increased ADH leads to renal water reabsorption and increased blood volume. b. Decreased kidney perfusion leads to renin released and increased angiotensin I. c. Increased renal sodium reabsorption leads to increased serum osmolality that leads to release of ADH. d. Increased angiotensin II leads to aldosterone secretion and results in increased renal sodium reabsorption. e. Increased angiotensin II leads to venous and arterial vasoconstriction, increased venous return to the heart, and increased BP.

B, D, E, C, A After the α- and β-adrenergic stimulation occurs in hypovolemic shock, angiotensin I is stimulated by decreased kidney perfusion and renin release. Increased angiotensin II leads to increased serum osmolality, the release of antidiuretic hormone (ADH), and aldosterone secretion that reabsorbs sodium. The increased ADH also increases renal water reabsorption, which increases blood volume.

A nurse witnesses a patient going into pulmonary edema. The patient exhibits respiratory distress, but the blood pressure is stable at this time. While waiting for help to arrive, the nurse performs the following actions in which order of priority? a. Rechecks vital signs b. Places the patient in high fowler's position c. Calls the respiratory therapy department for a ventilator d. Places the patient on a pulse oximeter and cardiac monitor e. Begins the patient's oxygen at 2 liters by nasal cannula as needed f. Administers the patient morphine sulfate intravenous injection as needed

B, E, D, A, F, C (D then A because intervention then reassessment!) MSC increases O2 supply by vasodilation!!!

A patient is being treated for cardiogenic shock. Which statement below best describes this condition? Select all that apply:*' A. "The patient will experience an increase in cardiac output due to an increase in preload and afterload." B. "A patient with this condition will experience decreased cardiac output and decreased tissue perfusion." C. "This condition occurs because the heart has an inadequate blood volume to pump." D. "Cardiogenic shock leads to pulmonary edema."

B. "A patient with this condition will experience decreased cardiac output and decreased tissue perfusion." D. "Cardiogenic shock leads to pulmonary edema."

You're precepting a new nurse. You ask the new nurse to list the purpose of why a patient with cardiogenic shock may benefit from an intra-aortic balloon pump. What responses below indicate the new nurse understands the purpose of an intra-aortic balloon pump? Select all that apply:* A. "This device increases the cardiac afterload, which will increase cardiac output." B. "This device will help increase blood flow to the coronary arteries." C. "The balloon pump will help remove extra fluid from the heart and lungs." D. "The balloon pump will help increase cardiac output."

B. "This device will help increase blood flow to the coronary arteries." D. "The balloon pump will help increase cardiac output."

A patient is on IV Norepinephrine for treatment of septic shock. Which statement is FALSE about this medication? A. "The nurse should titrate this medication to maintain a MAP of 65 mmHg or greater." B. "This medication causes vasodilation and decreases systemic vascular resistance." C. "It is used when fluid replacement is not unsuccessful." D. "It is considered a vasopressor."

B. "This medication causes vasodilation and decreases systemic vascular resistance."

If a patient has a blood volume of 5 Liters and loses 2 Liters, what is the percentage amount of volume loss this patient has experienced?* A. 25% B. 40% C. 30% D. 10%

B. 40%

You're working on a neuro unit. Which of your patients below are at risk for developing neurogenic shock? Select all that apply: A. A 36-year-old with a spinal cord injury at L4. B. A 42-year-old who has spinal anesthesia. C. A 25-year-old with a spinal cord injury above T6. D. A 55-year-old patient who is reporting seeing green halos while taking Digoxin.

B. A 42-year-old who has spinal anesthesia. C. A 25-year-old with a spinal cord injury above T6.

Select all the fluid types below that are considered colloids?* A. Fresh Frozen Plasma B. Albumin C. Normal Saline D. Lactated Ringer's E. Hetastarch F. Platelets

B. ALbumin E. Hetastarch

A 50-yr-old woman with a suspected brain tumor is scheduled for a CT scan with contrast media. The nurse notifies the physician that the patient reported an allergy to shellfish. Which response by the physician should the nurse question? A. Infuse IV diphenhydramine before the procedure. B. Administer lorazepam (Ativan) before the procedure. C. Complete the CT scan without the use of contrast media. D. Premedicate with hydrocortisone sodium succinate (Solu-Cortef).

B. Administer lorazepam (Ativan) before the procedure. An individual with an allergy to shellfish is at an increased risk to develop anaphylactic shock if contrast media is injected for a CT scan. To prevent anaphylactic shock, the nurse should always confirm the patient's allergies before diagnostic procedures (e.g., CT scan with contrast media). Appropriate interventions may include cancelling the procedure, completing the procedure without contrast media, or premedication with diphenhydramine or hydrocortisone. IV fluids may be given to promote renal clearance of the contrast media and prevent renal toxicity and acute kidney injury. The use of an antianxiety agent such as lorazepam would not be effective in preventing an allergic reaction to the contrast media.

A patient has a 10% loss of their blood volume. Select all the signs and symptoms this patient may present with?* A. Cool, clammy skin B. Blood pressure within normal limits C. Anxiety D. Capillary refill less than 2 seconds E. Urinary output greater than 30 mL/hr F. Mild tachycardia

B. Blood pressure within normal limits D. Capillary refill less than 2 seconds E. Urinary output greater than 30 mL/h

Your patient is started on an IV antibiotic to treat a severe infection. During infusion, the patient uses the call light to notify you that she feels a tight sensation in her throat and it's making it hard to breathe. You immediately arrive to the room and assess the patient. While auscultating the lungs you note wheezing. You also notice that the patient is starting to scratch the face and arms, and on closer inspection of the face you note redness and swelling that extends down to the neck and torso. The patient's vital signs are the following: blood pressure 89/62, heart rate 118 bpm, and oxygen saturation 88% on room air. You suspect anaphylactic shock. Select all the appropriate interventions for this patient: A. Slow down the antibiotic infusion B. Call a rapid response C. Place the patient on oxygen D. Prepare for the administration of Epinephrine

B. Call a rapid response C. Place the patient on oxygen D. Prepare for the administration of Epinephrine

Your patient, who is post-op from a kidney transplant, has developed septic shock. Which statement below best reflects the interventions you will perform for this patient? A. Administer Norepinephrine before attempting a fluid resuscitation. B. Collect cultures and then administer IV antibiotics. C. Check blood glucose levels before starting any other treatments. D. Administer Drotrecogin Alpha within 48-72 hours.

B. Collect cultures and then administer IV antibiotics.

A patient is in hypovolemic shock. Select all the stages that a patient can enter when in shock: A. Proliferative B. Compensatory C. Exudative D. Initial E. Progressive F. Fibrotic G. Refractory

B. Compensatory D. Initial E. Progressive G. Refractory

Your patient's blood pressure is 72/56, heart rate 126, and respiration 24. The patient has a fungal infection in the lungs. The patient also has a fever, warm/flushed skin, and is restless. You notify the physician who suspects septic shock. You anticipate that the physician will order what treatment FIRST? A. Low-dose corticosteroids B. Crystalloids IV fluid bolus C. Norepinephrine D. 2 units of Packed Red Blood Cells

B. Crystalloids IV fluid bolus

When caring for a critically ill patient who is being mechanically ventilated, the nurse will monitor for which clinical manifestation of multiple organ dysfunction syndrome (MODS)? A. Increased serum albumin B. Decreased respiratory compliance C. Increased gastrointestinal (GI) motility D. Decreased blood urea nitrogen (BUN)/creatinine ratio

B. Decreased respiratory compliance Clinical manifestations of MODS include symptoms of respiratory distress, signs and symptoms of decreased renal perfusion, decreased serum albumin and prealbumin, decreased GI motility, acute neurologic changes, myocardial dysfunction, disseminated intravascular coagulation (DIC), and changes in glucose metabolism.

You're providing education to a patient, who has a severe peanut allergy, on how to recognize the signs and symptoms of anaphylactic shock. Select all the signs and symptoms associated with anaphylactic shock: A. Hyperglycemia B. Difficulty speaking C. Feeling dizzy D. Hypertension E. Dyspnea F. Itchy G. Vomiting and Nausea H. Fever I. Slow heart rate

B. Difficulty speaking C. Feeling dizzy E. Dyspnea F. Itchy G. Vomiting and Nausea

A patient in neurogenic shock is ordered intravenous fluids due to severe hypotension. During administration of the fluids the nurse will monitor the patient closely and immediately report? A. Increase in blood pressure B. High central venous pressure (CVP) and pulmonary artery wedge pressure (PAWP) C. Urinary output of 300 mL in the past 5 hours D. Mean arterial pressure (MAP) 85 mmHg

B. High central venous pressure (CVP) and pulmonary artery wedge pressure (PAWP)

The nurse would recognize which clinical manifestation as suggestive of sepsis? A. Sudden diuresis unrelated to drug therapy B. Hyperglycemia in the absence of diabetes C. Respiratory rate of seven breaths per minute D. Bradycardia with sudden increase in blood pressure

B. Hyperglycemia in the absence of diabetes

The nurse would recognize which clinical manifestation as suggestive of sepsis? A. Sudden diuresis unrelated to drug therapy B. Hyperglycemia in the absence of diabetes C. Respiratory rate of seven breaths per minute D. Bradycardia with sudden increase in blood pressure

B. Hyperglycemia in the absence of diabetes Hyperglycemia in patients with no history of diabetes is a diagnostic criterion for sepsis. Oliguria, not diuresis, typically accompanies sepsis along with tachypnea and tachycardia.

During the __________ stage of shock, the signs and symptoms are very subtle. However, cells are experiencing _________ due to the lack of tissue perfusion, which causes the cells to switch from ___________ metabolism to _________ metabolism. A. Proliferative, hyperoxia, anaerobic, aerobic B. Initial, hypoxia, aerobic, anaerobic C. Compensatory, hypoxia, anaerobic, aerobic D. Fibrotic, hypoxia, aerobic, anaerobic

B. Initial, hypoxia, aerobic, anaerobic

Your patient in neurogenic shock is not responding to IV fluids. The patient is started on vasopressors. What option below, if found in your patient, would indicate the medication is working? A. Decreased CVP (central venous pressure) B. Mean arterial pressure (MAP) 90 mmHg C. Serum lactate 6 mmol/L D. Blood pH 7.20

B. Mean arterial pressure (MAP) 90 mmHg

A patient with cardiogenic shock has a blood pressure of 70/38. In addition, the patient is experiencing dyspnea with a respiratory rate of 32 breaths per minute and has an oxygen saturation of 82% on room air. On auscultation, you note crackles throughout the lung fields. You notify the physician. What order below would you ask for an order clarification?* A. Dopamine IV stat B. Normal saline IV bolus stat C. Furosemide IV stat D. Place patient on CPAP (continuous positive airway pressure)

B. Normal saline IV bolus stat

A patient in hypovolemic shock is receiving rapid infusions of crystalloid fluids. Which patient finding requires immediate nursing action?* A. Patient heart rate is 115 bpm B. Patient experiences dyspnea and crackles in lung fields C. Patient is anxious D. Patient's urinary output is 35 mL/hr

B. Patient experiences dyspnea and crackles in lung fields

Cardiac output is equal to the heart rate multiplied by the stroke volume. Treatment for cardiogenic shock includes medications that increase cardiac output. One of the factors that help determine cardiac output is stroke volume. Select all the factors that determine stroke volume?* A. Cardiac Index B. Preload C. Pulmonary capillary wedge pressure D. Afterload E. Heart rate F. Contractility

B. Preload D. Afterload F. Contractility

You're assessing your patient with cardiogenic shock, what signs and symptoms do you expect to find in this condition? Select all that apply:* A. Warm, flushed skin B. Prolonged capillary refill C. Urinary output >30 mL/hr D. Systolic blood pressure <90 mmHg E. Crackles in lung fields F. Dyspnea D. Decreased BUN and creatinine G. Strong peripheral pulses H. Chest pain

B. Prolonged capillary refill D. Systolic blood pressure <90 mmHg E. Crackles in lung fields F. Dyspnea H. Chest pain

A 64-yr-old woman is admitted to the emergency department vomiting bright red blood. The patient's vital signs are blood pressure of 78/58 mm Hg, pulse of 124 beats/min, respirations of 28 breaths/min, and temperature of 97.2°F (36.2°C). Which physician order should the nurse complete first? A. Obtain a 12-lead ECG and arterial blood gases. B. Rapidly administer 1000 mL normal saline solution IV. C. Administer norepinephrine (Levophed) by continuous IV infusion. D. Carefully insert a nasogastric tube and an indwelling bladder catheter.

B. Rapidly administer 1000 mL normal saline solution IV. Isotonic crystalloids, such as normal saline solution, should be used in the initial resuscitation of hypovolemic shock. Vasopressor drugs (e.g., norepinephrine) may be considered if the patient does not respond to fluid resuscitation and blood products. Other orders (e.g., insertion of nasogastric tube and indwelling bladder catheter and obtaining the diagnostic studies) can be initiated after fluid resuscitation is initiated.

What is the BEST position for a patient in anaphylactic shock? A. Lateral recumbent B. Supine with legs elevated C. High Fowler's D. Semi-Fowler's

B. Supine with legs elevated

You're assessing a patient's knowledge on how to use their EpiPen in case of an anaphylactic reaction. You're using an EpiPen trainer device to teach the patient. What demonstrated by the patient shows the patient knows how to administer the medication? Select all that apply: A. The patient injects the medication in the subq tissue of the abdomen. B. The patient massages the site after injection. C. The patient administers the injection through the clothes. D. The patient aspirates before injecting the medication.

B. The patient massages the site after injection. C. The patient administers the injection through the clothes.

A 42-year-old male patient is admitted with a spinal cord injury. The patient is experiencing severe hypotension and bradycardia. The patient is diagnosed with neurogenic shock. Why is hypotension occurring in this patient with neurogenic shock? A. The patient has an increased systemic vascular resistance. This increases preload and decreases afterload, which will cause severe hypotension. B. The patient's autonomic nervous system has lost the ability to regulate the diameter of the blood vessels and vasodilation is occurring. C. The patient's parasympathetic nervous system is being unopposed by the sympathetic nervous system, which leads to severe hypotension. D. The increase in capillary permeability has depleted the fluid volume in the intravascular system, which has led to severe hypotension.

B. The patient's autonomic nervous system has lost the ability to regulate the diameter of the blood vessels and vasodilation is occurring.

. You're caring for a patient with cardiogenic shock. Which finding below suggests the patient's condition is worsening? Select all that apply:* A. Blood pressure 95/68 B. Urinary output 20 mL/hr C. Cardiac Index 3.2 L/min/m2 D. Pulmonary artery wedge pressure 30 mmHg

B. Urinary output 20 mL/hr D. Pulmonary artery wedge pressure 30 mmHg

A patient with a severe infection has developed septic shock. The patient's blood pressure is 72/44, heart rate 130, respiration 22, oxygen saturation 96% on high-flow oxygen, and temperature 103.6 'F. The patient's mean arterial pressure (MAP) is 53 mmHg. Based on these findings, you know this patient is experiencing diminished tissue perfusion and needs treatment to improve tissue perfusion to prevent organ dysfunction. In regards to the pathophysiology of septic shock, what is occurring in the body that is leading to this decrease in tissue perfusion? Select all that apply: A. Absolute hypovolemia B. Vasodilation C. Increased capillary permeability D. Increased systemic vascular resistance E. Clot formation in microcirculation F. A significantly decreased cardiac output

B. Vasodilation C. Increased capillary permeability E. Clot formation in microcirculation

During anaphylactic shock the mast cells and basophils release large amounts of histamine. What effects does histamine have on the body during anaphylactic shock? Select all that apply: A. Decreases capillary permeability B. Vasodilation of vessels C. Decreases heart rate D. Shifts intravascular fluid to interstitial space E. Constricts the airways F. Stimulates contraction of GI smooth muscles H. Inhibits the production of gastric secretions I. Itching

B. Vasodilation of vessels D. Shifts intravascular fluid to interstitial space E. Constricts the airways F. Stimulates contraction of GI smooth muscles I. Itching

Select all the conditions below that increases a patient's risk for absolute hypovolemic shock:* A. Burns B. Vomiting C. Long bone fracture D. Surgery E. Diarrhea F. Sepsis

B. Vomiting D. Surgery E. Diarrhea

A patient with a fever is lethargic and has a blood pressure of 89/56. The patient's white blood cell count is elevated. The physician suspects the patient is developing septic shock. What other findings indicate this patient is in the "early" or "compensated" stage of septic shock? Select all that apply: A. Urinary output of 60 mL over 4 hours B. Warm and flushed skin C. Tachycardia D. Bradypnea

B. Warm and flushed skin C. Tachycardia

A patient is receiving large amounts of fluids for aggressive treatment of hypovolemic shock. The nurse makes it PRIORITY to?* A. Rapidly infuse the fluids B. Warm the fluids C. Change tubing in between bags D. Keep the patient supine

B. Warm the fluids

A patient is diagnosed with septic shock. As the nurse you know this is a __________ form of shock. In addition, you're aware that __________ and _________ are also this form of shock. A. obstructive; hypovolemic and anaphylactic B. distributive; anaphylactic and neurogenic C. obstructive; cardiogenic and neurogenic D. distributive; anaphylactic and cardiogenic

B. distributive; anaphylactic and neurogenic

A patient in the progressive stage of shock has rapid, deep respirations. The nurse determine that the patient's hyperventilation is compensating for a metabolic acidosis when the patient's ABG results include which of the following? a. pH 7.42, PaO2 80 b. pH 7.48, PaO2 69 c. pH 7.38, PaCO2 30 d. pH 7.32, PaCO2 48

C

A nurse has administered corticosteroids to a patient with multiple organ dysfunction syndrome (MODS). In what order should the results of this medication occur in this situation. (Answer with a letter followed by a comma and a space (e.g. A, B, C, D).) a. Renal excretion of potassium b. Increased intravascular volume c. Improved capillary permeability d. Enhanced renal artery perfusion

C, B, D, A Corticosteroids are administered to patients who have MODS to suppress the inflammatory response. If the patient responds positively to the corticosteroids, capillary permeability should improve, stopping leakage of intravascular fluid and helping restore intravascular blood volume. Improved intravascular volume should help improve the blood pressure, and if compensatory sympathetic discharge is halted, renal blood flow increases. Increased renal blood flow should increase the glomerular filtration rate and renal excretion of potassium.

Which interventions should be used for anaphylactic shock (SATA) a. antibiotics b. vasodilators c. antihistamines d. oxygen supplementation e. colloid volume expansion f. crystalloid volume expansion

C, D, E, F

Cardiac output is very important for determining if a patient is in cardiogenic shock. What is a normal cardiac output in an adult? .* A. 2-5 liters/minute B. 1-3 liters/minute C. 4-8 liters/minute D. 8-10 liters/minute

C. 4-8 liters/minute

As the nurse you know that in order for hypovolemic shock to occur the patient would need to lose __________ of their blood volume.* A. <30% B. >25% C. >15% D. >10%

C. >15%

A patient who is experiencing hypovolemic shock has decreased cardiac output, which contributes to ineffective tissue perfusion. The decrease in cardiac output occurs due to?* A. An increase in cardiac preload B. An increase in stroke volume C. A decrease in cardiac preload D. A decrease in cardiac contractility

C. A decrease in cardiac preload

A patient with neurogenic shock is experiencing a heart rate of 30 bpm. What medication does the nurse anticipate will be ordered by the physician STAT? A. Adenosine B. Warfarin C. Atropine D. Norepinephrine

C. Atropine

Which medications below are used in cardiogenic shock that provide a positive inotropic effect on the heart? Select all that apply:* A. Nitroglycerin B. Sodium Nitroprussidde C. Dobutamine D. Norepinephrine E. Dopamine

C. Dobutamine E. Dopamine

Which statements are INCORRECT about the compensatory stage of shock. Select all that apply: A. This stage is reversible. B. During this stage blood is shunted away from the kidneys, lungs, skin, and gastrointestinal system to the brain and heart. C. During this stage blood flow to the kidneys is reduced, which causes the kidneys to activate the renin-angiotensin system, and this will lead to major vasodilation to the arterial and venous system. D. One hallmark sign of this stage is that there is an increase in capillary permeability. E. A patient is at risk for a paralytic ileus during this stage.

C. During this stage blood flow to the kidneys is reduced, which causes the kidneys to activate the renin-angiotensin system, and this will lead to major vasodilation to the arterial and venous system. D. One hallmark sign of this stage is that there is an increase in capillary permeability.

A 35-year-old male arrives to the emergency room with multiple long bone fractures and an internal abdominal injury. The patient is anxious. Patient's vital signs are: Blood pressure 70/54, heart rate 125 bpm, respirations 30, oxygen saturation on 2 L nasal cannula 96%, temperature 99.3 'F, pain 6 on 1-10 scale. During assessment it is noted the skin is cool and clammy. The nurse will make it priority to?* A. Collect a urine sample B. Obtain an EKG C. Establish 2 large-bore IV access sites D. Place a warming blanket on the patient

C. Establish 2 large-bore IV access sites

The patient with hypovolemic shock is in need of clotting factors. Which type of fluid would best benefit this patient?* A. Platelets B. Albumin C. Fresh Frozen Plasma D. Packed Red Blood Cells

C. Fresh Frozen Plasma

One of your patients begins to vomit large amounts of bright red blood. The patient is taking Warfarin. You call a rapid response. Which assessment findings indicate this patient is developing hypovolemic shock? Select all that apply:* A. Temperature 104.8 'F B. Heart rate 40 bpm C. Heart rate 140 bpm D. Anxiety, restlessness E. Urinary output 15 mL/hr F. Blood pressure 70/56 G. Pale, cool skin H. Weak peripheral pulses I. Blood pressure 220/106

C. Heart rate 140 bpm D. Anxiety, restlessness E. Urinary output 15 mL/hr F. Blood pressure 70/56 G. Pale, cool skin H. Weak peripheral pulses

Your patient is having a sudden and severe anaphylactic reaction to a medication. You immediately stop the medication and call a rapid response. The patient's blood pressure is 80/52, heart rate 120, and oxygen saturation 87%. Audible wheezing is noted along with facial redness and swelling. As the nurse you know that the first initial treatment for this patient's condition is? A. IV Diphenhydramine B. IV Normal Saline Bolus C. IM Epinephrine D. Nebulized Albuterol

C. IM Epinephrine

A patient is in anaphylactic shock. The patient has a severe allergy to peanuts and mistakenly consumed an eggroll containing peanut ingredients during his lunch break. The patient is given Epinephrine intramuscularly. As the nurse, you know this medication will have what effect on the body? A. It will prevent a recurrent attack. B. It will cause vasoconstriction and decrease the blood pressure. C. It will help dilate the airways. D. It will help block the effects of histamine in the body.

C. It will help dilate the airways.

You're providing care to a patient experiencing neurogenic shock due to an injury at T4. As the nurse, you know which of the following is a patient safety priority? A. Keeping the head of the bed greater than 45 degrees at all times. B. Repositioning the patient every thirty minutes. C. Keeping the patient's spine immobilized. D. Avoiding log-rolling the patient during transport.

C. Keeping the patient's spine immobilized.

The nurse is caring for a 29-yr-old man who was admitted 1 week ago with multiple rib fractures, pulmonary contusions, and a left femur fracture from a motor vehicle crash. The attending physician states the patient has developed sepsis, and the family members have many questions. Which information should the nurse include when explaining the early stage of sepsis? A. Antibiotics are not useful when an infection has progressed to sepsis. B. Weaning the patient away from the ventilator is the top priority in sepsis. C. Large amounts of IV fluid are required in sepsis to fill dilated blood vessels. D. The patient has recovered from sepsis if he has warm skin and ruddy cheeks.

C. Large amounts of IV fluid are required in sepsis to fill dilated blood vessels. Patients with sepsis may be normovolemic, but because of acute vasodilation, relative hypovolemia and hypotension occur. Patients in septic shock require large amounts of fluid replacement and may require frequent fluid boluses to maintain circulation. Antibiotics are an important component of therapy for patients with septic shock. They should be started after cultures (e.g., blood, urine) are obtained and within the first hour of septic shock. Oxygenating the tissues is the top priority in sepsis, so efforts to wean septic patients from mechanical ventilation halt until sepsis is resolving. Additional respiratory support may be needed during sepsis. Although cool and clammy skin is present in other early shock states, the patient in early septic shock may feel warm and flushed because of a hyperdynamic state.

A patient's localized infection has become systemic and septic shock is suspected. What medication is expected to treat septic shock refractory to fluids? A. Insulin infusion B. Furosemide (Lasix) IV push C. Norepinephrine administered by titration D. Administration of nitrates and β-adrenergic blockers

C. Norepinephrine administered by titration If fluid resuscitation using crystalloids is not effective, vasopressor medications such as norepinephrine (Levophed) and dopamine are indicated to restore mean arterial pressure (MAP). Nitrates and β-adrenergic blockers are most often used in the treatment of patients in cardiogenic shock. Furosemide (Lasix) is indicated for patients with fluid volume overload. Insulin infusion may be administered to normalize blood sugar and improve overall outcomes, but it is not considered a medication used to treat shock.

The nurse is caring for a 72-yr-old man in cardiogenic shock after an acute myocardial infarction. Which clinical manifestations would be most concerning? A. Restlessness, heart rate of 124 beats/min, and hypoactive bowel sounds B. Mean arterial pressure of 54 mm Hg; increased jaundice; and cold, clammy skin C. PaO2 of 38 mm Hg, serum lactate level of 46.5 mcg/dL, and puncture site bleeding D. Agitation, respiratory rate of 32 breaths/min, and serum creatinine of 2.6 mg/dL

C. PaO2 of 38 mm Hg, serum lactate level of 46.5 mcg/dL, and puncture site bleeding Severe hypoxemia, lactic acidosis, and bleeding are clinical manifestations of the irreversible state of shock. Recovery from this stage is not likely because of multiple organ system failure. Restlessness, tachycardia, and hypoactive bowel sounds are clinical manifestations that occur during the compensatory stage of shock. Decreased mean arterial pressure, jaundice, cold and clammy skin, agitation, tachypnea, and increased serum creatinine are clinical manifestations of the progressive stage of shock.

A patient who has cardiogenic shock is experiencing labored breathing and low oxygen levels. A STAT chest x-ray is ordered. The x-ray results show pulmonary edema. The physician orders Furosemide IV. What finding would require immediate nursing action? A. Blood pressure 98/54 B. Urinary output 45 mL/hr C. Potassium 1.8 mEq/L D. Heart rate 110 bpm

C. Potassium 1.8 mEq/L

Based on this scenario, what stage of shock is this patient most likely experiencing: A 74-year-old patient is extremely confused and does not respond to commands or stimulation. The patient respiratory rate is 28 and labored, oxygen saturation 86%, heart rate 120, blood pressure 70/40, mean arterial pressure is 50 mmHg, and temperature is 97 'F. The patient's heart rhythm is atrial fibrillation. The patient's urinary output is 5 mL/hr. The patient's labs: blood pH 7.15, serum lactate 15 mmol/L, BUN 55 mg/dL, Creatinine 6 mg/dL. In addition, the patient is now starting to have slight oozing of blood around puncture sites.* A. Initial B. Proliferative C. Progressive D. Compensatory

C. Progressive

You're caring for a patient who is experiencing shock. Which lab result below demonstrates that the patient's cells are using anaerobic metabolism? A. Ammonia 18 µ/dL B. Potassium 4.5 mEq/L C. Serum Lactate 9 mmol/L D. Bicarbonate 23 mEq/L

C. Serum Lactate 9 mmol/L

A patient is having an anaphylactic reaction to an IV medication. What is the FIRST action the nurse should take? A. Administer Epinephrine B. Call a Rapid Reponse C. Stop the medication D. Administer a breathing treatment

C. Stop the medication

13. They physician orders a Dobutamine IV drip on a patient in cardiogenic shock. After starting the IV drip, the nurse would make it priority to monitor for?* A. Rebound hypertension B. Ringing in the ears C. Worsening hypotension D. Severe headache

C. Worsening hypotension

A patient in shock has a nursing diagnosis of fear related to severity of condition and perceived threat of death as manifested by verbalization of anxiety about condition and fear of death. What is an appropriate nursing intervention for this patient? a. administer antianxiety agents b. allow caregivers to visit as much as possible c. call a member of the clergy to visit the patient d. inform the patient of the current plan of care and its rationale

D

What mechanism that can trigger SIRS is related to myocardial infarction or pancreatitis? a. abscess formation b. microbial invasion c. global perfusion deficits d. ischemic or necrotic tissue

D

Which statement describing SIRS and/or MODS is accurate? a. MODS may occur independently from SIRS b. All patients with septic shock develop MODS c. the GI system is often the first to show evidence of dysfunction in SIRS and MODS d. A common initial mediator that causes endothelial damage leading to SIRS and MODS is endotoxin.

D

During what stage (or class) of hypovolemic shock does the sympathetic nervous system attempt to maintain cardiac output?* A. I B. III C. IV D. II

D, II

A patient in septic shock is experiencing hyperglycemia. The patient is started on an insulin drip. A blood glucose goal for this patient would be: A. <110 mg/dL B. <80 mg/dL C. >200 mg/dL D. <180 mg/dL

D. <180 mg/dL

Which patient below is at MOST risk for developing cardiogenic shock?* A. A 52-year-old male who is experiencing a severe allergic reaction from shellfish. B. A 25-year-old female who has experienced an upper thoracic spinal cord injury. C. A 72-year-old male who is post-op from a liver transplant. D. A 49-year-old female who is experiencing an acute myocardial infarction.

D. A 49-year-old female who is experiencing an acute myocardial infarction.

The nurse is assisting in the care of several patients in the critical care unit. Which patient is most at risk for developing multiple organ dysfunction syndrome (MODS)? A. A 22-yr-old patient with systemic lupus erythematosus admitted with a pelvic fracture B. A 48-yr-old patient with lung cancer admitted for syndrome of inappropriate antidiuretic hormone and hyponatremia C. A 65-yr-old patient with coronary artery disease, dyslipidemia, and primary hypertension admitted for unstable angina D. A 82-yr-old patient with type 2 diabetes mellitus and chronic kidney disease admitted for peritonitis related to a peritoneal dialysis catheter infection

D. A 82-yr-old patient with type 2 diabetes mellitus and chronic kidney disease admitted for peritonitis related to a peritoneal dialysis catheter infection A patient with peritonitis is at high risk for developing sepsis. In addition, a patient with diabetes is at high risk for infections and impaired healing. Sepsis and septic shock are the most common causes of MODS. Individuals at greatest risk for developing MODS are older adults and persons with significant tissue injury or preexisting disease. MODS can be initiated by any severe injury or disease process that activates a massive systemic inflammatory response.

. _____________ is the force the heart has to pump against to get blood out of the ventricle.* A. Cardiac output B. Cardiac index C. Preload D. Afterload

D. Afterload

A patient is at risk for septic shock when a microorganism invades the body. Which microorganism is the MOST common cause of sepsis? A. Fungus B. Virus C. Parasite D. Bacteria

D. Bacteria

During what stage of shock does the body attempt to utilize the hormonal, neural, and biochemical responses of the body? A. Refractory B. Initial C. Proliferative D. Compensatory

D. Compensatory

What laboratory finding is consistent with a medical diagnosis of cardiogenic shock? A. Decreased liver enzymes B. Increased white blood cells C. Decreased red blood cells, hemoglobin, and hematocrit D. Increased blood urea nitrogen (BUN) and serum creatinine (Cr) levels

D. Increased blood urea nitrogen (BUN) and serum creatinine (Cr) levels The renal hypoperfusion that accompanies cardiogenic shock results in increased BUN and creatinine levels. Impaired perfusion of the liver results in increased liver enzymes, but white blood cell levels do not typically increase in cardiogenic shock. Red blood cell indices are typically normal because of relative hypovolemia.

A massive gastrointestinal bleed has resulted in hypovolemic shock in an older patient. What is a priority nursing diagnosis? A. Acute pain B. Impaired skin integrity C. Decreased cardiac output D. Ineffective tissue perfusion

D. Ineffective tissue perfusion The many deleterious effects of shock are all related to inadequate perfusion and oxygenation of every body system. This nursing diagnosis supersedes the other diagnoses.

A patient in septic shock receives large amounts of IV fluids. However, this was unsuccessful in maintaining tissue perfusion. As the nurse, you would anticipate the physician to order what NEXT? A. IV corticosteroids B. Colloids C. Dobutamine D. Norepinephrine

D. Norepinephrine

During what stage of shock is the body unable to compensate for tissue perfusion and the body's cell start to experience hypoxic injury that result in __________capillary permeability? A. Refractory, increased B. Exudative, decreased C. Compensatory, increased D. Progressive, increased

D. Progressive, increased

After coronary artery bypass graft surgery a patient has postoperative bleeding that requires returning to surgery for repair. During surgery, the patient has a myocardial infarction (MI). After restoring the patient's body temperature to normal, which patient parameter is the most important for planning nursing care? A. Cardiac index (CI) of 5 L/min/m2 B. Central venous pressure of 8 mm Hg C. Mean arterial pressure (MAP) of 86 mm Hg D. Pulmonary artery pressure (PAP) of 28/14 mm Hg

D. Pulmonary artery pressure (PAP) of 28/14 mm Hg Pulmonary hypertension as indicated by an elevated PAP indicates impaired forward flow of blood because of left ventricular dysfunction or hypoxemia. Both can be a result of the MI. The CI, CVP, and MAP readings are normal.

Which stage of shock is irreversible and unmanageable? A. Progressive B. Initial C. Exudative D. Refractory

D. Refractory

During shock, when a patient experiences a drop in cardiac output, the body tries to compensate by stimulating the sympathetic nervous system, which causes the release of _________ and ________. This will lead to? A. acetylcholine and dopamine, vasodilation B. epinephrine and norepinephrine, vasodilation C. dopamine and epinephrine, vasoconstriction D. norepinephrine and epinephrine, vasoconstriction

D. norepinephrine and epinephrine, vasoconstriction

8. You're providing care to a patient in anaphylactic shock. What is NOT a typical medical treatment for this condition, and if ordered the nurse should ask for an order clarification? A. IV Diphenhydramine B. Epinephrine C. Corticosteroids D. Isotonic intravenous fluids E. IV Furosemide

E. IV Furosemide

. True or False: If a patient with cardiogenic shock is given a medication that will decrease cardiac afterload, it will decrease stroke volume.* True False

FALSE

TRUE or FALSE: All types of shock during the compensatory (early) stage will cause a patient to experience cold and clammy (moist or sweaty) skin.

FALSE

True or False: A patient with acute pancreatitis is presenting with Turner and Cullen's Sign. This patient is at risk for absolute hypovolemic shock.* True False

FALSE

True or False: Hypovolemic shock occurs where there is low fluid volume in the interstitial compartment.* True False

FALSE

True or False: Septic shock causes system wide vasodilation which leads to an increase in systemic vascular resistance. In addition, septic shock causes increased capillary permeability and clot formation in the microcirculation throughout the body.

FALSE

True or False: The parasympathetic nervous system loses the ability to stimulate nerve impulses in patients who are experiencing neurogenic shock. This leads to hemodynamic changes.

FALSE

a nurse si caring for a client who has DIC. which labs indicate the client's clotting factors are depleted SATA PLT 100 fibrinogen levels 120 fibrin degradation products 4.3 d-dimer 0.03 sedimentation rate 38mm/hr

PLT 100 fibrinogen levels 120

The nurse is caring for a patient with multiple organ dysfunction syndrome (MODS) who exhibits signs of cardiovascular dysfunction. Which interventions are appropriate for volume management in the patient? Select all that apply. Select all that apply Providing volume replacement therapy Decreasing the preload by administering diuretics Monitoring arterial pressure-based cardiac output (APCO) Using central venous catheter for hemodynamic monitoring Maintaining mean arterial pressure at greater than 45 mm Hg

Providing volume replacement therapy Monitoring arterial pressure-based cardiac output (APCO) Using central enous catheter for hemodynamic monitoring

Which findings indicate the development of acute respiratory distress syndrome (ARDS) in a patient with systemic inflammatory response syndrome (SIRS)? Select all that apply. Pulmonary hypertension Decreased lung compliance Decreased minute ventilation Abnormally slow breathing rate Bilateral diffuse infiltrates in the chest

Pulmonary hypertension Decreased lung compliance Bilateral diffuse infiltrates in the chest

A diabetic patient being cared for following amputation of the left lower leg has large amounts of purulent drainage from the stump and redness and edema around the stump wound. The patient is lethargic and confused. Vital signs include a blood pressure of 80/60 mm Hg, respiratory rate of 32 breaths/minute, and pulse of 112 beats/minute. Which signs and symptoms would support a diagnosis of multiple organ dysfunction syndrome (MODS)? Select all that apply Pulse of 112 beats/minute Lethargy and confusion Respiratory rate of 32 breathes/minute Purulent drainage from stump Blood pressure of 80/60 mm Hg Edema and redness of stump wound

Pulse of 112 beats/minute Lethargy and confusion Respiratory rate of 32 breathes/minute Blood pressure of 80/60 mm Hg

a nurse is caring for a group of clients. whos at risk for obstructive shock a client with occasional PVCs on the ECG monitor a client who has been experiencing vomiting and diarrhea for several days a clieent who has a gram negative bacterial infection a client who has pulmonary arterial stenosis

a client who has pulmonary arterial stenosis

What physical problems could precipitate hypovolemic shock (select all that apply)? a. Burns b. Ascites c. Vaccines d. Insect bites e. Hemorrhage f. Ruptured spleen

a, b, e, f. Hypovolemic shock occurs when there is a loss of intravascular fluid volume from fluid loss (as in hemorrhage or severe vomiting and diarrhea), fluid shift (as in burns or ascites), or internal bleeding (as with a ruptured spleen). Vaccines and insect bites would precipitate the anaphylactic type of distributive shock.

The patient is in the compensatory stage of shock. What manifestations indicate this to the nurse (select all that apply)? a. Pale and cool b. Unresponsive c. Lower BP than baseline d. Moist crackles in the lungs e. Hyperactive bowel sounds f. Tachypnea and tachycardia

a, c, f. In the compensatory stage of shock the patient's skin will be pale and cool (α-adrenergic stimulation). There may also be a change in level of consciousness but the person will be responsive, the BP will be lower than baseline, bowel sounds will be hypoactive (α-adrenergic stimulation), and tachypnea and tachycardia (β-adrenergic stimulation) will occur. Unresponsiveness and moist crackles in the lungs occur in the progressive stage of shock.

In the compensatory stage of hypovolemic shock, to what organs does blood flow decrease after the sympathetic nervous system activates the α-adrenergic stimulation (select all that apply)? a. Skin b. Brain c. Heart d. Kidneys e. Gastrointestinal tract

a, d, e. After sympathetic nervous system activation of vasoconstriction, blood flow to nonvital organs, such as skin, kidneys, and the gastrointestinal (GI) tract is diverted or shunted to the most essential organs of the heart and the brain. The patient will feel cool and clammy, the renin- angiotensin-aldosterone system will be activated, and the patient may develop a paralytic ileus.

When caring for a patient in cardiogenic shock, the nurse recognizes that the metabolic demands of turning and moving the patient exceed the oxygen supply when what change is revealed in hemodynamic monitoring? a. SvO2 from 62% to 54% b. CO from 4.2 L/min to 4.8 L/min c. Stroke volume (SV) from 52 to 68 mL/beat d. SVR from 1300 dyne/sec/cm5 to 1120 dyne/sec/cm5

a. A decreased mixed venous oxygen saturation (SvO2) indicates that the patient has used the venous oxygen reserve and is at greater risk for anaerobic metabolism. The SvO2 decreases when more oxygen is used by the cells, as in activity or hypermetabolism. All of the other values indicate an improvement in the patient's condition.

A patient with hypovolemic shock is receiving lactated Ringer's solution for fluid replacement therapy. During this therapy, which laboratory result is most important for the nurse to monitor? a. Serum pH b. Serum sodium c. Serum potassium d. Hemoglobin (Hgb) and hematocrit (Hct)

a. Lactated Ringer's solution may increase lactate levels, which a damaged liver cannot convert to bicarbonate. This may intensify the metabolic lactic acidosis that occurs in progressive shock, necessitating careful attention to the patient's acid-base balance. Sodium and potassium levels as well as hemoglobin (Hgb) and hematocrit (Hct) levels should be monitored in all patients receiving fluid replacement therapy.

A 70-year-old patient is malnourished, has a history of type 2 diabetes mellitus, and is admitted from the nursing home with pneumonia. For which kind of shock should the nurse closely monitor this patient? a. Septic shock b. Neurogenic shock c. Cardiogenic shock d. Anaphylactic shock

a. Older adults with chronic diseases and malnourished or debilitated patients are at risk of developing septic shock, especially when they have an infection (e.g., pneumonia, urinary tract infection) or indwelling lines or catheters.

A common manifestation of the rapid progression of sepsis is: a. Tachycardia b. Hypertension c. Polyuria d. Bradypnea

a. Tachycardia

Appropriate treatment modalities for the management of cariogenic shock include (select all that apply): a. dobutamine to increase myocardial contractility b. vasopressors to increase systemic vascular resistance c. circulatory assist devices such as an intraortic balloon pump d. corticosteroids to stabilize the cell wall in the infarcted myocardium e. trendelenburg position to facilitate venous return and increased preload

a. dobutamine to increase myocardial contractility c. circulatory assist devices such as an intraortic balloon pump

a nurse is planning care for a client who has septic shock. which of the following actions is the priority for the nurse to take maintain adequate fluid volume with IV infusions administer ABX Monitor hemodynamic status administer vasopressor medication

administer ABX

A patient with severe trauma has been treated for hypovolemic shock. The nurse recognizes that the patient is in the irreversible stage of shock when what is included in assessment findings? a. A lactic acidosis with a pH of 7.32 b. Marked hypotension and refractory hypoxemia c. Unresponsiveness that responds only to painful stimuli d. Profound vasoconstriction with absent peripheral pulses

b. During both the compensatory and the progressive stages of shock, the sympathetic nervous system is activated in an attempt to maintain CO and SVR. In the irreversible stage of shock, the sympathetic nervous system can no longer compensate to maintain homeostasis and a loss of vasomotor tone leading to profound hypotension affects perfusion to all vital organs, causing increasing cellular hypoxia, metabolic acidosis, and cellular death.

Which hemodynamic monitoring description of the identified shock is accurate? a. Tachycardia with hypertension is characteristic of neurogenic shock. b. In cardiogenic shock the patient will have an increased pulmonary artery wedge pressure (PAWP) and a decreased cardiac output (CO). c. Anaphylactic shock is characterized by increased systemic vascular resistance (SVR), decreased CO, and decreased PAWP. d. In septic shock, bacterial endotoxins cause vascular changes that result in increased SVR and decreased CO.

b. Hemodynamic monitoring in cardiogenic shock will reveal increased pulmonary artery wedge pressure (PAWP) and decreased CO. The characteristic signs of neurogenic shock are bradycardia and hypotension. Septic shock manifests with decreased systemic vascular resistance (SVR) and increased CO. Hypovolemic shock is characterized by increased SVR, decreased CO, and decreased PAWP.

What abnormal finding should the nurse expect to find in early compensatory shock? a. Metabolic acidosis b. Increased serum sodium c. Decreased blood glucose d. Increased serum potassium

b. In early compensatory shock, activation of the renin- angiotensin-aldosterone system stimulates the release of aldosterone, which causes sodium reabsorption and potassium excretion by the kidney, elevating serum sodium levels and decreasing serum potassium levels. Blood glucose levels are elevated during the compensatory stage of shock in response to catecholamine stimulation of the liver, which releases its glycogen stores in the form of glucose. Metabolic acidosis does not occur until the progressive stage of shock. At this stage compensatory mechanisms become ineffective and anaerobic cellular metabolism causes lactic acid production.

When shock occurs in a patient with pulmonary embolism or abdominal compartment syndrome, what type of shock would that be? a. Distributive shock b. Obstructive shock c. Cardiogenic shock d. Hypovolemic shock

b. Obstructive shock occurs when a physical obstruction impedes the filling or outflow of blood, resulting in reduced cardiac output (CO).

The nurse determines that a large amount of crystalloid fluids administered to a patient in septic shock is effective when hemodynamic monitoring reveals what? a. CO of 2.6 L/min b. CVP of 15 mm Hg c. PAWP of 4 mm Hg d. Heart rate (HR) of 106 bpm

b. The endpoint of fluid resuscitation in septic and hypovolemic shock is a central venous pressure (CVP) of 15 mm Hg or a PAWP of 10-12mmHg. The CO is too low and the heart rate is too high to indicate adequate fluid replacement.

The nurse suspects sepsis as a cause of shock when the laboratory test results indicate a. hypokalemia. b. thrombocytopenia. c. decreased hemoglobin. d. increased blood urea nitrogen (BUN)

b. Thrombocytopenia can occur. When sepsis is the cause of shock, endotoxin stimulates a cascade of inflammatory responses that start with the release of tumor necrosis factor (TNF) and interleukin-1 (IL-1), which stimulate other inflammatory mediators. The release of platelet- activating factor causes formation of microthrombi and vessel obstruction. There is vasodilation, increased capillary permeability, and neutrophil and platelet aggregation and adhesion to the endothelium. The process does not occur in other types of shock until late stages of shock.

78 y.o man has confusion and temperature of 104. He's diabetic with purulent drainage from his right heel. After infusion of 3L of NS, his assessment findings are BP 84/40, HR 110, Res 42 and shallow, CO 8L/min and PAWP 4mm. Patient is most likely in: a. sepsis b. septic shock c. multiple organ dysfunction syndrome d. SIRS

b. septic shock

Priority Decision: What is the priority nursing responsibility in the prevention of shock? a. Frequently monitoring all patients' vital signs b. Using aseptic technique for all invasive procedures c. Being aware of the potential for shock in all patients at risk d. Teaching patients health promotion activities to prevent shock

c. Prevention of shock necessitates identification of persons who are at risk and a thorough baseline nursing assessment with frequent ongoing assessments to monitor and detect changes in patients at risk. Frequent monitoring of all patients' vital signs is not necessary. Aseptic technique for all invasive procedures should always be implemented but will not prevent all types of shock. Health promotion activities that reduce the risk for precipitating conditions, such as coronary artery disease or anaphylaxis, may help to prevent shock in some cases.

When administering any vasoactive drug during the treatment of shock, the nurse should know that what is the goal of the therapy? a. Increasing urine output to 50 mL/hr b. Constriction of vessels to maintain BP c. Maintaining a MAP of at least 60 mm Hg d. Dilating vessels to improve tissue perfusion

c. Vasoactive drugs are those that can either dilate or constrict blood vessels and are used in various stages of shock treatment. When using either vasodilators or vasoconstrictors, it is important to maintain a mean arterial pressure (MAP) of at least 60 mm Hg so that adequate perfusion is maintained. The other goals would be appropriate only with either vasodilators or vasoconstrictors, not with all vasoactive drugs.

a nurse is caring for a client who has a prescription for an afterload reducing medication. the nurse should identify that this medication is administered for which of the following types of shock cardiogenic hypovolemic obstructive distributive

cardiogenic

The nurse is caring for a patient in septic shock. Which hemodynamic change would the nurse expect? a)Increased ejection fraction. b)Increased mean arterial pressure. c)Decreased central venous pressure. d)Decreased systemic vascular resistance.

d) decreased systemic vascular resistance

The nurse is caring for a critically ill patient. The nurse suspects that the patient has progressed beyond the compensatory stage of shock if what occurs? a)Increased blood glucose levels b)Increased serum sodium levels c)Increased serum calcium levels d)Increased serum potassium levels

d) increased serum potassium levels

A massive gastrointestinal bleed has resulted in hypovolemic shock in an older patient. What is a priority nursing diagnosis? A. Acute pain B. Impaired tissue integrity C. Decreased cardiac output D. Ineffective tissue perfusion

d) ineffective tissue perfusion

When assessing a patient in shock, the nurse recognizes that the hemodynamics of shock include? a)Normal cardiac output and normal blood pressure. b)Increase in central venous pressure in hypovolemic shock. c)Increase in systemic vascular resistance in septic shock d)Variations in cardiac output and decreased systemic vascular resistance in septic shock.

d)variation in cardiac output and decreased systemic vascular resistance in septic shock

What is the key factor in describing any type of shock? a. Hypoxemia b. Hypotension c. Vascular collapse d. Inadequate tissue perfusion

d. Although all of the factors may be present, regardless of the cause, the end result is inadequate supply of oxygen and nutrients to body cells from inadequate tissue perfusion.

As the body continues to try to compensate for hypovolemic shock, there is increased angiotensin II from the activation of the renin-angiotensin-aldosterone system. What physiologic change occurs related to the increased angiotensin II? a. Vasodilation b. Decreased blood pressure (BP) and CO c. Aldosterone release results in sodium and water excretion d. Antidiuretic hormone (ADH) release increases water reabsorpton

d. Angiotensin II vasoconstricts both arteries and veins, which increases blood pressure (BP). It stimulates aldosterone release from the adrenal cortex, which results in sodium and water reabsorption and potassium excretion by the kidneys. The increased sodium raises serum osmolality and stimulates the pituitary gland to release antidiuretic hormone (ADH), which increases water reabsorption, which further increases blood volume, leading to an increase in BP and CO.

During administration of IV norepinephrine (Levophed), what should the nurse assess the patient for? a. Hypotension b. Marked diuresis c. Metabolic alkalosis d. Decreased tissue perfusion

d. As a vasopressor, norepinephrine may cause severe vasoconstriction, which would further decrease tissue perfusion, especially if fluid replacement is inadequate. Vasopressors generally cause hypertension, reflex bradycardia, and decreased urine output because of decreased renal blood flow. They do not directly affect acid-base balance.

Progressive tissue hypoxia leading to anaerobic metabolism and metabolic acidosis is characteristic of the progressive stage of shock. What changes in the heart contribute to this increasing tissue hypoxia? a. Arterial constriction causes decreased perfusion. b. Vasoconstriction decreases blood flow to pulmonary capillaries. c. Increased capillary permeability and profound vasoconstriction cause increased hydrostatic pressure. d. Decreased perfusion occurs, leading to dysrhythmias, decreased CO, and decreased oxygen delivery to cells.

d. Decreased myocardial perfusion leads to dysrhythmias and myocardial ischemia, further decreasing CO and oxygen delivery to cells. The kidney's renin-angiotensin- aldosterone system activation causes arteriolar constriction that decreases perfusion. In the lung, vasoconstriction of arterioles decreases blood flow and a ventilation-perfusion mismatch occurs. Areas of the lung that are oxygenated are not perfused because of the decreased blood flow, resulting in hypoxemia and decreased oxygen for cells. Increased capillary permeability and vasoconstriction cause increased hydrostatic pressure that contributes to the fluid shifting to interstitial spaces.

Priority Decision: A patient with acute pancreatitis is experiencing hypovolemic shock. Which initial orders for the patient will the nurse implement first? a. Start 1000 mL of normal saline at 500 mL/hr. b. Obtain blood cultures before starting IV antibiotics. c. Draw blood for hematology and coagulation factors. d. Administer high-flow oxygen (100%) with a non-rebreather mask.

d. In every type of shock there is a deficiency of oxygen to the cells and high-flow oxygen therapy is indicated. Fluids would be started next, blood cultures would be done before any antibiotic therapy, and laboratory specimens then could be drawn

In late irreversible shock in a patient with massive thermal burns, what should the nurse expect the patient's laboratory results to reveal? a. Respiratory alkalosis b. Decreased potassium c. Increased blood glucose d. Increased ammonia (NH3) levels

d. In late irreversible shock, progressive cellular destruction causes changes in laboratory findings that indicate organ damage. Increasing ammonia levels indicate impaired liver function. Metabolic acidosis is usually severe as cells continue anaerobic metabolism and the respiratory alkalosis that may occur in the compensatory stage has failed to compensate for the acidosis. Potassium levels increase and blood glucose decreases.

Which intervention is recommended when sepsis is suspected and a patient is awaiting transfer to the ICU? a. Fever reduction b. Hemodynamic monitoring c. Pain management d. Oxygen administration

d. Oxygen administration

Patient has a SCI at T4, vital signs include falling BP with bradycardia. Nurse recognizes the patient is experiencing: a. relative hypovolemia b. absolute hypovolemia c. neurogenic shock from low blood flow d. neurogenic shock from massive vasodilation

d. neurogenic shock from massive vasodilation

urse is caring for a client who is experiencing wheezing and swelling of the tongue. which of the following medications should the nurse anticipate administering first methylprednisolone diphenhydramine epinephrine dobutamine

epinephrine

a nurse is assessing a client and suspects the client is experiencing DIC. which of the following physical findings should the nurse anticipate bradycardia hypertension epistaxis xerostomia

epistaxis

a nurse is caring for a client who has DIC. which meds should the nurse anticipate giving heparin vitamin k mefoxin simvastatin

heparin

a nurse is teaching a newly licensed nurse about HIT. which of the following frisk factors for this disorder should the nurse include in the teaching warfarin therapy for a fib placental abruption SLE heparin therapy for DVT

heparin therapy for DVT

a nurse in the ED is assessing a client who is in shock. what are the expected findings SATA HR 60bpm seizure activity respiratory rate 42/min increased urinary output weak and thready pulse

seizure activity respiratory rate 42/min weak and thready pulse

Which intervention may prevent GI bacterial and endotoxin translocation in a critically ill patient with SIRS? a. early enteral feedings b. surgical removal of necrotic tissue c. aggressive multiple antibiotic therapy d. strict aseptic technique in all procedures

A

Which indicators of tissue perfusion should be monitored in critically ill patients by the nurse (SATA) a. skin b. urine output c. level of conciousness d. activities of daily living e. vital signs, including pulse oximetry f. peripheral pulses with capillary refill

A, B, C, E, F

The patient is admitted with an unusual infection. The nurse knows that a mechanical tissue trauma that can trigger SIRS will not occur with this patient because what types of injuries cause a mechanical tissue trauma trigger of SIRS? (SATA) a. burns b. fungi c. viruses d. crush injuries e. surgical procedures

A, D, E

A patient has lost 750 mL of blood volume. The MD orders Normal Saline infusion. Using the 3:1 rule, how much crystalloid solution should be prescribed by the doctor?* A. 2,250 mL of Normal Saline B. 250 mL of Normal Saline C. 375 mL of Normal Saline C. 1,225 mL of Normal Saline

A. 2,250 mL of Normal Saline

You're providing an in-service to new nurse graduates on the fluid treatment for hypovolemic shock. You ask the participants to list the types of crystalloid solutions used in hypovolemic shock. Which responses are INCORRECT? Select all that apply:* A. Albumin B. Lactated Ringer's C. Normal Saline D. Hetastarch

A. Albumin D. Hetastarch

What is the MOST important step a nurse can take to prevent anaphylactic shock in a patient? A. Assessing, documenting, and avoiding all the patient allergies B. Administering Epinephrine C. Administering Corticosteroids D. Establishing IV access

A. Assessing, documenting, and avoiding all the patient allergies

You receive a patient in the ER who has sustained a cervical spinal cord injury. You know this patient is at risk for neurogenic shock. What hallmark signs and symptoms, if experienced by this patient, would indicate the patient is experiencing neurogenic shock? Select all that apply: A. Blood pressure 69/38 B. Heart rate 170 bpm C. Blood pressure 250/120 D. Heart rate 29 E. Warm and dry extremities F. Cool and clammy extremities G. Temperature 104.9 'F H. Temperature 95 'F

A. Blood pressure 69/38 D. Heart rate 29 E. Warm and dry extremities H. Temperature 95 'F

Your patient, who is post-op from a gastrointestinal surgery, is presenting with a temperature of 103.6 'F, heart rate 120, blood pressure 72/42, increased white blood cell count, and respirations of 21. An IV fluid bolus is ordered STAT. Which findings below indicate that the patient is progressing to septic shock? Select all that apply: A. Blood pressure of 70/34 after the fluid bolus B. Serum lactate less than 2 mmol/L C. Patient needs Norepinephrine to maintain a mean arterial pressure (MAP) greater than 65 mmHg despite fluid replacement D. Central venous pressure (CVP) of 18

A. Blood pressure of 70/34 after the fluid bolus C. Patient needs Norepinephrine to maintain a mean arterial pressure (MAP) greater than 65 mmHg despite fluid replacement

A patient is receiving treatment for an acute myocardial infarction. The nurse is closely monitoring the patient for signs and symptoms associated with cardiogenic shock. Which value below is associated with cardiogenic shock? A. Cardiac index 1.5 L/min/m2 B. Pulmonary capillary wedge pressure (PCWP) 10 mmHg C. Central venous pressure (CVP) 4 mmHg D. Troponin <0.01 ng/mL

A. Cardiac index 1.5 L/min/m2

When the body is attempting to compensate for shock the adrenal cotex will release aldosterone due to the presence of angiotension II. Select all the effects adosterone will have on the body to increase cardiac output and maintain tissue perfusion: SATA A. Increase blood volume B. Causes the kidneys to keep sodium and water C. Causes the kidneys to excrete sodium and water D. Cause the urine to have a low osmolality

A. Increase blood volume B. Causes the kidneys to keep sodium and water

When caring for a patient in acute septic shock, what should the nurse anticipate? A. Infusing large amounts of IV fluids B. Administering osmotic and/or loop diuretics C. Administering IV diphenhydramine (Benadryl) D. Assisting with insertion of a ventricular assist device (VAD)

A. Infusing large amounts of IV fluids Septic shock is characterized by a decreased circulating blood volume. Volume expansion with the administration of IV fluids is the cornerstone of therapy. The administration of diuretics is inappropriate. VADs are useful for cardiogenic shock not septic shock. Diphenhydramine may be used for anaphylactic shock but would not be helpful with septic shock.

In neurogenic shock, a patient will experience a decrease in tissue perfusion. This deprives the cells of oxygen that make up the tissues and organs. Select all the mechanisms, in regards to pathophysiology, of why this is occurring: A. Loss of vasomotor tone B. Increase systemic vascular resistance C. Decrease in cardiac preload D. Increase in cardiac afterload E. Decrease in venous blood return to the heart F. Venous blood pooling in the extremities

A. Loss of vasomotor tone C. Decrease in cardiac preload E. Decrease in venous blood return to the heart F. Venous blood pooling in the extremities

You're providing care to a patient who has experienced a 45% loss of their fluid volume and is experiencing hypovolemic shock. The patient has hemodynamic monitoring and fluid resuscitation is being attempted. Which finding indicates the patient is still in hypovolemic shock?* A. Low central venous pressure B. High pulmonary artery wedge pressure C. Elevated mean arterial pressure D. Low systemic vascular resistance

A. Low central venous pressure

Your patient is receiving aggressive treatment for septic shock. Which findings demonstrate treatment is NOT being successful? Select all that apply: A. MAP (mean arterial pressure) 40 mmHg B. Urinary output of 10 mL over 2 hours C. Serum Lactate 15 mmol/L D. Blood glucose 120 mg/dL E. CVP (central venous pressure) less than 2 mmHg

A. MAP (mean arterial pressure) 40 mmHg B. Urinary output of 10 mL over 2 hours C. Serum Lactate 15 mmol/L E. CVP (central venous pressure) less than 2 mmHg

A patient is 1 hour post-op from abdominal surgery and had lost 20% of their blood volume during surgery. The patient is experiencing signs and symptoms of hypovolemic shock. What position is best for this patient?* A. Modified Trendelenburg B. Trendelenburg C. High Fowler's D. Supine

A. Modified Trendelenburg

You're developing a nursing plan of care for a patient with neurogenic shock. As the nurse, you know that due to venous blood pooling from vasodilation a deep vein thrombosis can occur in this type of shock. A patient goal is that the patient will be free from the development of a deep vein thrombosis. Select all the nursing interventions below that can help the patient meet this goal: A. Perform range of motion exercises daily. B. Place a pillow underneath the patient knees as needed. C. Administer anticoagulants as scheduled per physician's order. D. Apply compression stockings daily.

A. Perform range of motion exercises daily. C. Administer anticoagulants as scheduled per physician's order. D. Apply compression stockings daily.

________________ is the amount the ventricle stretches at the end of diastole.* A. Preload B. Afterload C. Stroke Volume D. Contractility

A. Preload

The physician orders a patient in septic shock to receive a large IV fluid bolus. How would the nurse know if this treatment was successful for this patient? A. The patient's blood pressure changes from 75/48 to 110/82. B. Patient's CVP 2 mmHg C. Patient's skin is warm and flushed. D. Patient's urinary output is 20 mL/hr.

A. The patient's blood pressure changes from 75/48 to 110/82.

Which statement is true about colloid solutions? Select all that apply:* A. These solutions are made up of large molecules that cannot diffuse through the capillary wall, so more fluid stays in the intravascular space longer when compared with the action of a crystalloid solution. B. These solutions can diffuse through the capillary wall so less fluid stays in the intravascular system when compared to the action of a crystalloid solution. C. The nurse should monitor for an anaphylactic reaction when these products are administered. D. These fluids are considered hypertonic solutions.

A. These solutions are made up of large molecules that cannot diffuse through the capillary wall, so more fluid stays in the intravascular space longer when compared with the action of a crystalloid solution. C. The nurse should monitor for an anaphylactic reaction when these products are administered.

A 25-year-old female is admitted to the ER in anaphylactic shock due to a bee sting. According to the patient's mother, the patient is severely allergic to bees and was recently stung by one. This type of anaphylactic reaction is known as a? A. Type I Hypersensivity Reaction B. Type II Hypersensivity Reaction C. Type III Hypersensivity Reaction D. Type IV Hypersensivity Reaction

A. Type I Hypersensivity Reaction

A patient with cardiogenic shock has an intra-aortic balloon pump. As the nurse you know that during ________ the balloon deflates and during _____ the balloon inflates in a section of the aorta.* A. systole, diastole B. diastole, systole C. inspiration, expiration D. expiration, inspiration

A. systole, diastole

Anaphylactic shock can occur due to either an immunological or non-immunological cause. Select ALL the CORRECT statements about the differences between an immunological reaction (anaphylactic) and non-immunological reaction (anaphylactoid): A. "In an immunological reaction (anaphylactic) IgE antibodies are created and they attach to mast cells and basophils." B. "An immunological reaction (anaphylactic) requires a patient to be sensitized for anaphylactic shock to occur." C. "A non-immunological reaction (anaphylactoid) causes the same reaction as an anaphylactic reaction, but it's not due to immunoglobulin IgE antibodies." D. "Some common substances that cause a non-immunological reaction (anaphylactoid) are IV contrast dyes and NSAIDS." E. "A patient does not have to be sensitized for a non-immunological reaction (anaphylactoid) to occur and it can happen with first time exposure."

ALL

Select all the complications that can arise from the progressive stage of shock: A. Acute respiratory distress syndrome B. Extreme edema C. Elevated ammonia and lactate levels D. GI bleeding and ulcers E. Dysrhythmias F. Myocardial infraction G. Acute tubular necrosis H. Disseminated intravascular clotting

ALL

1. A 198-lb patient is to receive a dobutamine infusion at 5 mcg/kg/minute. The label on the infusion bag states: dobutamine 250 mg in 250 mL normal saline. When setting the infusion pump, the nurse will set the infusion rate at how many mL per hour?

ANS: 27 In order to administer the dobutamine at the prescribed rate of 5 mcg/kg/minute from a concentration of 250 mg in 250 mL, the nurse will need to infuse 27 mL/hour. DIF: Cognitive Level: Apply (application) REF: 1598 TOP: Nursing Process: Implementation

1. The health care provider orders the following interventions for a 67-kg patient who has septic shock with a BP of 70/42 mm Hg and oxygen saturation of 90% on room air. In which order will the nurse implement the actions?(Put a comma and a space between each answer choice [A, B, C, D, E].) Obtain blood and urine cultures. Give vancomycin (Vancocin) 1 g IV. Start norepinephrine (Levophed) 0.5 mcg/min. Infuse normal saline 2000 mL over 30 minutes. Titrate oxygen administration to keep O2 saturation >95%.

ANS: E, D, C, A, B The initial action for this hypotensive and hypoxemic patient should be to improve the oxygen saturation, followed by infusion of IV fluids and vasopressors to improve perfusion. Cultures should be obtained before administration of antibiotics. DIF: Cognitive Level: Analyze (analysis) REF: 1601 OBJ: Special Questions: Prioritization TOP: Nursing Process: Implementation

A patient's ABG results include pH 7.31, Paco2 50 mm Hg, Pao2 51 mm Hg, and HCO3 24 mEq/L. Oxygen is administered at 2 L/min, and the patient is placed in high-Fowler's position. An hour later, the ABGs are repeated with results of pH 7.36, Paco2 40 mm Hg, Pao2 60 mm Hg, and HCO3 24 mEq/L. What is most important for the nurse to do? a. Increase the oxygen flow rate to 4 L/min. b. Document the findings in the patient's record. c. Reposition the patient in a semi-Fowler's position. d. Prepare the patient for endotracheal intubation and mechanical ventilation.

ANS: A

11. Norepinephrine (Levophed) has been prescribed for a patient who was admitted with dehydration and hypotension. Which patient data indicate that the nurse should consult with the health care provider before starting the norepinephrine? a. The patients central venous pressure is 3 mm Hg. b. The patient is in sinus tachycardia at 120 beats/min. c. The patient is receiving low dose dopamine (Intropin). d. The patient has had no urine output since being admitted.

ANS: A Adequate fluid administration is essential before administration of vasopressors to patients with hypovolemic shock. The patients low central venous pressure indicates a need for more volume replacement. The other patient data are not contraindications to norepinephrine administration. DIF: Cognitive Level: Apply (application) REF: 1600 TOP: Nursing Process: Implementation

15. A patient with septic shock has a BP of 70/46 mm Hg, pulse 136, respirations 32, temperature 104 F, and blood glucose 246 mg/dL. Which intervention ordered by the health care provider should the nurse implement first? a. Give normal saline IV at 500 mL/hr. b. Give acetaminophen (Tylenol) 650 mg rectally. c. Start insulin drip to maintain blood glucose at 110 to 150 mg/dL. d. Start norepinephrine (Levophed) to keep systolic blood pressure >90 mm Hg.

ANS: A Because of the low systemic vascular resistance (SVR) associated with septic shock, fluid resuscitation is the initial therapy. The other actions also are appropriate, and should be initiated quickly as well. DIF: Cognitive Level: Apply (application) REF: 1600 OBJ: Special Questions: Prioritization TOP: Nursing Process: Implementation

1. A 78-kg patient with septic shock has a urine output of 30 mL/hr for the past 3 hours. The pulse rate is 120/minute and the central venous pressure and pulmonary artery wedge pressure are low. Which order by the health care provider will the nurse question? a. Give PRN furosemide (Lasix) 40 mg IV. b. Increase normal saline infusion to 250 mL/hr. c. Administer hydrocortisone (Solu-Cortef) 100 mg IV. d. Titrate norepinephrine (Levophed) to keep systolic BP >90 mm Hg.

ANS: A Furosemide will lower the filling pressures and renal perfusion further for the patient with septic shock. The other orders are appropriate. DIF: Cognitive Level: Apply (application) REF: 1596 TOP: Nursing Process: Implementation

19. During change-of-shift report, the nurse is told that a patient has been admitted with dehydration and hypotension after having vomiting and diarrhea for 4 days. Which finding is most important for the nurse to report to the health care provider? a. New onset of confusion b. Heart rate 112 beats/minute c. Decreased bowel sounds d. Pale, cool, and dry extremities

ANS: A The changes in mental status are indicative that the patient is in the progressive stage of shock and that rapid intervention is needed to prevent further deterioration. The other information is consistent with compensatory shock. DIF: Cognitive Level: Apply (application) REF: 1595 OBJ: Special Questions: Prioritization TOP: Nursing Process: Assessment

14. Which data collected by the nurse caring for a patient who has cardiogenic shock indicate that the patient may be developing multiple organ dysfunction syndrome (MODS)? a. The patients serum creatinine level is elevated. b. The patient complains of intermittent chest pressure. c. The patients extremities are cool and pulses are weak. d. The patient has bilateral crackles throughout lung fields.

ANS: A The elevated serum creatinine level indicates that the patient has renal failure as well as heart failure. The crackles, chest pressure, and cool extremities are all consistent with the patients diagnosis of cardiogenic shock. DIF: Cognitive Level: Apply (application) REF: 1605 TOP: Nursing Process: Assessment

18. A patient is admitted to the emergency department (ED) for shock of unknown etiology. The first action by the nurse should be to a. administer oxygen. b. obtain a 12-lead electrocardiogram (ECG). c. obtain the blood pressure. d. check the level of consciousness.

ANS: A The initial actions of the nurse are focused on the ABCsairway, breathing, and circulationand administration of oxygen should be done first. The other actions should be accomplished as rapidly as possible after oxygen administration. DIF: Cognitive Level: Apply (application) REF: 1597 OBJ: Special Questions: Prioritization TOP: Nursing Process: Implementation

2. A nurse is caring for a patient with shock of unknown etiology whose hemodynamic monitoring indicates BP 92/54, pulse 64, and an elevated pulmonary artery wedge pressure. Which collaborative intervention ordered by the health care provider should the nurse question? a. Infuse normal saline at 250 mL/hr. b. Keep head of bed elevated to 30 degrees. c. Hold nitroprusside (Nipride) if systolic BP <90 mm Hg. d. Titrate dobutamine (Dobutrex) to keep systolic BP >90 mm Hg.

ANS: A The patients elevated pulmonary artery wedge pressure indicates volume excess. A saline infusion at 250 mL/hr will exacerbate the volume excess. The other actions are appropriate for the patient. DIF: Cognitive Level: Apply (application) REF: 1589 TOP: Nursing Process: Planning

A patient with a history of alcoholism is admitted to the intensive care unit (ICU) with hemorrhage from esophageal varices. Admission vital signs are blood pressure 84/58 mm Hg, pulse 105 beats/min, and respiratory rate 32 breaths/min. The nurse recognizes the onset of systemic inflammatory response syndrome (SIRS) upon finding: a. Pulmonary edema. b. Cardiac dysrhythmias. c. Absent bowel sounds. d. Decreasing blood pressure.

ANS: A This is the FIRST SIGN of SIRS.


Conjuntos de estudio relacionados

Data Visualization Midterm -- Tableau Questions

View Set

7. Cloud Application Security (This one was hard) 80%

View Set